Answer

问题及解答

[Homework] 3.2

Posted by haifeng on 2020-11-02 16:38:39 last update 2020-11-02 16:38:39 | Edit | Answers (2)

P. 118  习题 3.2


1.  利用洛必达法则求下列极限:

(9)    $\lim\limits_{x\rightarrow\infty}x(e^{\frac{2}{x}}-1)$

 

 

(11)    $\lim\limits_{x\rightarrow 1^-}(1-x)^{\cos\frac{\pi}{2}x}$

 

 

 

 

3.   设 $f(x)=\begin{cases}\frac{g(x)}{x}, & x\neq 0,\\ 0, & x=0 ,\end{cases}$  其中 $g(x)$ 具有二阶导数, 并且 $g(0)=g'(0)=0$, $g''(0)=a$,  求 $f'(0)$.

 

 

1

Posted by haifeng on 2020-11-02 20:37:24

1. (9)

\[
\begin{split}
\text{原式}&=\lim_{x\rightarrow\infty}\frac{e^{\frac{2}{x}}-1}{\frac{1}{x}}\\
&\xlongequal[\frac{0}{0}]{\text{洛}}\lim_{x\rightarrow\infty}\frac{e^{\frac{2}{x}}\cdot\frac{-2}{x^2}}{-\frac{1}{x^2}}\\
&=\lim_{x\rightarrow\infty}2e^{\frac{2}{x}}\\
&=2
\end{split}
\]

 

 


(11)

 

\[
\begin{split}
\text{原式}&=\lim_{x\rightarrow 1^-}e^{\cos\frac{\pi}{2}x\cdot\ln(1-x)}\\
&=e^{\lim\limits_{x\rightarrow 1^-}\frac{\ln(1-x)}{\sec\frac{\pi}{2}x}}\\
\end{split}
\]

我们计算指数部分, 其属于 $\frac{\infty}{\infty}$ 类型, 应用洛必达法则

\[
\begin{split}
&\lim_{x\rightarrow 1^-}\frac{\ln(1-x)}{\sec\frac{\pi}{2}x}\\
=&\lim_{x\rightarrow 1^-}\frac{\frac{1}{1-x}\cdot(-1)}{\sec\frac{\pi}{2}x\cdot\tan\frac{\pi}{2}x\cdot\frac{\pi}{2}}\\
=&\lim_{x\rightarrow 1^-}\frac{2}{\pi}\cdot\frac{\cos\frac{\pi}{2}x\cdot\cos\frac{\pi}{2}x}{(x-1)\cdot\sin\frac{\pi}{2}x}\\
=&\lim_{x\rightarrow 1^-}\frac{2\cos^2\frac{\pi}{2}x}{\pi\cdot(x-1)}\\
=&\lim_{x\rightarrow 1^-}\frac{1+\cos\pi x}{\pi\cdot(x-1)}\\
=&\lim_{x\rightarrow 1^-}\frac{-\sin(\pi x)\cdot\pi}{\pi}\\
=&0
\end{split}
\]

因此, 原极限等于 $e^0=1$.

2

Posted by haifeng on 2020-11-04 11:43:38

3.

\[
\begin{split}
f'(0)&=\lim_{x\rightarrow 0}\frac{f(x)-f(0)}{x-0}\\
&=\lim_{x\rightarrow 0}\frac{\frac{g(x)}{x}-0}{x}\\
&=\lim_{x\rightarrow 0}\frac{g(x)}{x^2}
\end{split}
\]

由于 $g'(0)$ 存在, 故 $g(x)$ 在 $x=0$ 处连续, 从而 $\lim\limits_{x\rightarrow 0}g(x)=g(0)$. 因此, 上面属于 $\frac{0}{0}$ 型, 可以使用洛必达法则.

\[
\begin{split}
\text{上式}&=\lim_{x\rightarrow 0}\frac{g'(x)}{2x}\\
&=\frac{1}{2}\cdot\lim_{x\rightarrow 0}\frac{g'(x)-g'(0)}{x-0}\\
&=\frac{1}{2}\cdot g''(0)\\
&=\frac{1}{2}a
\end{split}
\]

即 $f'(0)=\frac{a}{2}$.


 

我们看一下另一种做法是否有问题.

当 $x\neq 0$ 时,

\[
f'(x)=(\frac{g(x)}{x})'=\frac{g'(x)\cdot x-g(x)\cdot 1}{x^2}
\]

由于 $g(x)$ 具有二阶导数, 故 $g'(x)$ 是连续函数, 因此 $f'(x)=\frac{xg'(x)-g(x)}{x^2}$ 也连续, 从而

\[
\begin{split}
f'(0)&=\lim_{x\rightarrow 0}f'(x)=\lim_{x\rightarrow 0}\frac{xg'(x)-g(x)}{x^2}\\
&=\lim_{x\rightarrow 0}\frac{1\cdot g'(x)+x\cdot g''(x)-g'(x)}{2x}\\
&=\lim_{x\rightarrow 0}\frac{x\cdot g''(x)}{2x}\\
&=\lim_{x\rightarrow 0}\frac{1}{2}g''(x)\\
&=\frac{1}{2}g''(0)\\
&=\frac{1}{2}a.
\end{split}
\]

请指出这里存在的错误.